User avatar
 
ManhattanPrepLSAT1
Thanks Received: 1909
Atticus Finch
Atticus Finch
 
Posts: 2851
Joined: October 07th, 2009
 
This post thanked 2 times.
 
 

Re: Q18 - Columnist: Taking a strong position

by ManhattanPrepLSAT1 Fri Dec 31, 1999 8:00 pm

This is a tough principle question. We're asked to find a principle to which the argument conforms. The conclusion is that it's best not to take a strong position unless one has considered all the evidence. The evidence for this position is that in order to understand an issue, one must consider all the evidence. The first sentence is not part of the argument core - notice the pivot with the word "but."

We can put this argument into conditional logic in the following way:

P: U --> CE
-------------
C: SP --> CE

The assumption being: SP --> U

Notation Key: U - understand, CE - consider the evidence, SP - take a strong position

Answer choice (C) provides the assumption in the form of a contrapositive.

Incorrect Answers
(A) represents a reversal of the assumption.
(B) connects related and important terms of the argument, but not in a way that would bridge the gap.
(D) connects an important term with an irrelevant term.
(E) connects an important term with an irrelevant term.


#officialexplanation
 
bradleygirard
Thanks Received: 17
Forum Guests
 
Posts: 27
Joined: May 12th, 2010
 
 
trophy
First Responder
 

Q18 - Columnist: Taking a strong position

by bradleygirard Sun May 23, 2010 2:34 pm

Any help that I can get with this one would be greatly appreciated. I can easily see why (B)(D) and (E) can be nixed as answer choices, and I can understand why (C) is a good choice, but I don't understand why (A) is not. The best explanation that
I can come up with is that (a) is a mistaken reversal, but it is all made a bit more confusing with the addition of 'reasonable'.
Thanks.
 
Nina
Thanks Received: 0
Atticus Finch
Atticus Finch
 
Posts: 103
Joined: October 15th, 2012
 
 
 

Re: Q18 - Columnist: Taking a strong position

by Nina Wed Mar 20, 2013 10:39 pm

hey Matt,

thanks for your explanation. but do we treat this principle support question here as a "necessary assumption" question?

Thanks!
User avatar
 
ManhattanPrepLSAT1
Thanks Received: 1909
Atticus Finch
Atticus Finch
 
Posts: 2851
Joined: October 07th, 2009
 
This post thanked 2 times.
 
 

Re: Q18 - Columnist: Taking a strong position

by ManhattanPrepLSAT1 Thu Mar 21, 2013 5:44 pm

Yeah, the language is more similar to a Necessary Assumption question than to a Sufficient Assumption question. And I think I know where you're going with your question. Were you about to ask whether answer choice (C) is more of a Sufficient Assumption answer choice?

The answer is yes, but even on Necessary Assumption questions there many instances when the correct answer simply bridges a gap in a chain of conditional reasoning. Find the pieces that need to be connected and then find the answer choice that does so.

Additionally, we don't want to be too strict with the answer choice, since the question stem asks for the principle that the argument most closely conforms to.

Does that address your concern on this one?
 
callie44
Thanks Received: 0
Forum Guests
 
Posts: 6
Joined: March 15th, 2013
 
 
 

Re: Q18 - Columnist: Taking a strong position

by callie44 Sun May 26, 2013 1:26 pm

I don't understand why the right answer to this principle question is an answer that fills the gap. So, I'm not confused between sufficient & necessary assumptions, but why are we looking for an assumption in general? Also, could you explain why the wrong answer choices are wrong? Thanks
 
dean.won
Thanks Received: 4
Forum Guests
 
Posts: 46
Joined: January 25th, 2013
 
 
 

Re: Q18 - Columnist: Taking a strong position

by dean.won Sat Jun 08, 2013 1:52 am

mattsherman Wrote:This is a tough principle question. We're asked to find a principle to which the argument conforms. The conclusion is that it's best not to take a strong position unless one has considered all the evidence. The evidence for this position is that in order to understand an issue, one must consider all the evidence. The first sentence is not part of the argument core - notice the pivot with the word "but."

We can put this argument into conditional logic in the following way:

P: U --> CE
-------------
C: SP --> CE

The assumption being: SP --> U

Notation Key: U - understand, CE - consider the evidence, SP - take a strong position

Answer choice (C) provides the assumption in the form of a contrapositive.

Incorrect Answers
(A) represents a reversal of the assumption.
(B) connects related and important terms of the argument, but not in a way that would bridge the gap.
(D) connects an important term with an irrelevant term.
(E) connects an important term with an irrelevant term.


matt,

how is consider evidence impartially = consider all important info

i dont understand how the CE part in the premise and conclusion are the same

i also dont understand how this is a principle support (assumption) question

thx
 
samantha.b233
Thanks Received: 2
Vinny Gambini
Vinny Gambini
 
Posts: 11
Joined: January 04th, 2013
 
This post thanked 1 time.
 
 

Re: Q18 - Columnist: Taking a strong position

by samantha.b233 Sun Jun 09, 2013 8:59 pm

I am confused. But I don't think this is an assumption family question. This is not a principle support question, in my opinion. It mentions "conform" instead of "support."

A is wrong because the question stem is about when a person should not take a strong position. We really don't know a lot about when they should.

B is wrong because, the question stem (conclusion) is about when not to take strong position, how can we impartially consider evidence is not relevant here.

C is correct because it is in line with the question stem. It is true that the question stem states:

Premise: U -> CE (~CE ->~U)
Conclusion: ~CE -> SP

C says: ~U -> ~SP. This is in line with the question stem.

D is wrong because question stem does not tell us why we should understand an issue fully.

E is wrong because the "only if" is really not mentioned in the question stem.
 
vania_apple
Thanks Received: 0
Forum Guests
 
Posts: 5
Joined: June 06th, 2013
 
 
 

Re: Q18 - Columnist: Taking a strong position

by vania_apple Sat Jun 15, 2013 4:06 am

This is my take on A:

If you try negating the last sentence in the stimulus it should run along the lines of: if one has already considered all important evidence conflicting with that position, then it may be reasonable/you could take a strong position on an issue. (correct me if i'm wrong)

Answer choice A mentions 'evidence regarding that issue..', however, in order for the negation to kick, you must consider all important evidence conflicting with that position. A doesn't mention that.
 
amil91
Thanks Received: 5
Elle Woods
Elle Woods
 
Posts: 59
Joined: August 02nd, 2013
 
 
 

Re: Q18 - Columnist: Taking a strong position

by amil91 Mon Nov 11, 2013 4:57 pm

vania_apple Wrote:This is my take on A:

If you try negating the last sentence in the stimulus it should run along the lines of: if one has already considered all important evidence conflicting with that position, then it may be reasonable/you could take a strong position on an issue. (correct me if i'm wrong)

Answer choice A mentions 'evidence regarding that issue..', however, in order for the negation to kick, you must consider all important evidence conflicting with that position. A doesn't mention that.

Your logic here is wrong. You cannot simply negate the last sentence as it is a conditional statement. You must negate and reverse. The last sentence is saying If one has NOT already considered.... then it is best NOT to take a strong position. The only logically sound inference one can make from this is: If it is best to take a strong position then one has already considered... Choice A places the necessary component on the sufficient side.

While it is good you nixed the answer for a different reason, your logic was not correct.
 
timsportschuetz
Thanks Received: 45
Elle Woods
Elle Woods
 
Posts: 95
Joined: June 30th, 2013
 
This post thanked 6 times.
 
trophy
First Responder
 

Re: Q18 - Columnist: Taking a strong position

by timsportschuetz Wed Nov 13, 2013 8:10 pm

As mentioned above, the first sentence is irrelevant to the argument core. However, I would like to add another reason in addition to the one already discussed (That the "but" in the second sentence is an indicator of a language shift).

I think it is more important to realize that sentence 1 is NOT a conditional statement! The LSAT has been doing this alot lately... they will write sentences that SOUND like conditional logic, however, they are not. Since sentence 1 uses ambiguous and overly weak language ("makes one likely..."), we CANNOT infer any conditional logic from the same! Whenever a sentence says that something "can/may/could/perhaps/even if/possibility", NO CONDITIONAL LOGIC IS PRESENT! How could stating that something "may" occur indicate sufficiency and/or necessity? It never can!
 
meltgonzalez
Thanks Received: 0
Forum Guests
 
Posts: 2
Joined: October 17th, 2013
 
 
 

Re: Q18 - Columnist: Taking a strong position

by meltgonzalez Sun Nov 24, 2013 8:05 pm

yes!
I think timsportschuetz is absolutely correct. In fact that is why this is a principles question. (C) is correct specifically because it is a broader statement, a principle, to which the stimulus conforms. We have no idea when it is "reasonable" to take a strong position. We cannot simply assume it is reasonable to do so because we know, as the stimulus states, that we shouldn't do it if we haven't already considered all important evidence conflicting with that position. Just because we have considered the evidence, doesn't necessarily make anything reasonable. It might make it reasonable to YOU, but "reasonable" is an evaluative statement that has not been defined in the passage so one should not jump to that conclusion and more importantly, it cannot really function as a principle because it wouldn't necessarily suggest anything.

Sadly I chose A, but I think i secretly knew that C was the right answer ha.
 
pkhosraviani
Thanks Received: 0
Forum Guests
 
Posts: 2
Joined: November 25th, 2013
 
 
 

Re: Q18 - Columnist: Taking a strong position

by pkhosraviani Mon Nov 25, 2013 4:53 pm

My difficulty is in understanding the question stem. The answer seems to fill in a gap, like an assumption family question would require, but the stem is not an assumption family stem. Can anyone help with this?
 
timsportschuetz
Thanks Received: 45
Elle Woods
Elle Woods
 
Posts: 95
Joined: June 30th, 2013
 
 
trophy
First Responder
 

Re: Q18 - Columnist: Taking a strong position

by timsportschuetz Mon Nov 25, 2013 5:04 pm

This is a principle question. Specifically, you are looking for an answer that will act as a "CLOUD" or "ENVELOPE" that encompasses the original argument and will allow the original argument to "fit" inside of the principle as stated in the correct answer choice. Keep in mind that the correct answer choice will usually be more general and less restrictive in nature in this types of questions. Let me provide you with a simplistic example of your task:

Argument: Since Charlie is a spider monkey, he likes to play in trees.

Correct answer choice: If an animal is a monkey, then that animal likes to play in trees.

Please let me know if this helps...
 
cwolfington
Thanks Received: 4
Jackie Chiles
Jackie Chiles
 
Posts: 29
Joined: May 15th, 2014
 
This post thanked 1 time.
 
 

Re: Q18 - Columnist: Taking a strong position

by cwolfington Wed Aug 06, 2014 6:38 pm

The big difference between (A) and (C) is that (A) uses the word "reasonable". The stimulus says nothing about being reasonable or unreasonable, so (C) is the better answer.
 
asafezrati
Thanks Received: 6
Atticus Finch
Atticus Finch
 
Posts: 116
Joined: December 07th, 2014
 
 
 

Re: Q18 - Columnist: Taking a strong position

by asafezrati Wed Dec 17, 2014 6:14 am

Hey,

The explanations above are great. Now that I understand this question I kinda like it. :D

Here are my top points:
1. The "but" in the second sentence really takes the first sentence out of the logical equations.
2. The question stem asks for a principle which exist in the reasoning. Correct me if I'm wrong, but it seems that there are several principles at hand, and not just the one that appears as an unstated assumption.

Awesome!
 
RodrigoA220
Thanks Received: 1
Vinny Gambini
Vinny Gambini
 
Posts: 6
Joined: October 22nd, 2017
 
 
 

Re: Q18 - Columnist: Taking a strong position

by RodrigoA220 Fri Jan 12, 2018 10:08 pm

ManhattanPrepLSAT1 Wrote:This is a tough principle question. We're asked to find a principle to which the argument conforms. The conclusion is that it's best not to take a strong position unless one has considered all the evidence. The evidence for this position is that in order to understand an issue, one must consider all the evidence. The first sentence is not part of the argument core - notice the pivot with the word "but."

We can put this argument into conditional logic in the following way:

P: U --> CE
-------------
C: SP --> CE

The assumption being: SP --> U

Notation Key: U - understand, CE - consider the evidence, SP - take a strong position

Answer choice (C) provides the assumption in the form of a contrapositive.

Incorrect Answers
(A) represents a reversal of the assumption.
(B) connects related and important terms of the argument, but not in a way that would bridge the gap.
(D) connects an important term with an irrelevant term.
(E) connects an important term with an irrelevant term.


#officialexplanation


Can you explain why the assumption is SP -> U and not U-SP? Wouldn't U-->SP take us from the premise to the conclusion? Whereas SP--->U would take us from the conclusion to the premise?
User avatar
 
ohthatpatrick
Thanks Received: 3806
Atticus Finch
Atticus Finch
 
Posts: 4661
Joined: April 01st, 2011
 
This post thanked 2 times.
 
 

Re: Q18 - Columnist: Taking a strong position

by ohthatpatrick Sun Jan 14, 2018 7:24 pm

It's not always about going from the Conclusion to the Premise (although I understand you've probably heard that a million times).

If a Conclusion is a conditional statement, then your goal is just to create the logic conveyor belt that the conclusion is selling.

If the Conclusion is
A ---> Z

and we're given
A --> B

then we need
B ---> Z

==== meanwhile ======
If the Conclusion is
A ---> Z

and we're given
Y --> Z

then we need
A ---> Y


So for Q18,
If the Conclusion is
C: SP --> CE

And we're given
P: U --> CE

Then we need
SP --> U

Because we're trying to get from SP to CE.
SP --> U ---> CE

If we had
U --> SP
and
U --> CE

that wouldn't give us a way to get from SP to CE.
Hope this helps.
 
YudeS218
Thanks Received: 0
Vinny Gambini
Vinny Gambini
 
Posts: 21
Joined: September 01st, 2017
 
 
 

Re: Q18 - Columnist: Taking a strong position

by YudeS218 Tue Dec 25, 2018 7:31 am

I chose the right answer, but really don't understand why the first sentence is not part of the argument core - because of the pivotal word "but"?

Why there is a but and the sentence before but would be irrelevant?

Thanks in advance!